Economist: Unemployment will soon decrease. If total government spending significantly increases next year, the econo...

Jimmywantstogotolawschool on April 21, 2020

Why A not C?

Can you explain this inference?

Reply
Create a free account to read and take part in forum discussions.

Already have an account? log in

Annie on April 23, 2020

Hi @Jimmywantstogotolawschool,

This question asks you to pick the answer choice which allows the conclusion to be "properly inferred." Essentially, this means you are looking for a premise that is being assumed by the argument. Here's a breakdown of the argument:

Premise: If gov. spending significantly increases next year, the economy will be stimulated, and unemployment will decrease.
Premise: If gov. spending significantly decreases next year, businesses will retain earnings and employ workers, and unemployment will decrease.
Premise: ???
Conclusion: Unemployment will soon decrease.

When you get questions asking what the argument is assuming, it's sometimes best to try to come up with the answer before turning to the answer choices. Here we are given two scenarios of what government spending will do next year and then given the conclusion that unemployment will decrease. But, what about all the other scenarios of what could happen with government spending next year? It could stay the same as this year, increase only a little, or decrease only a little. We don't know what will happen with unemployment in those circumstances. So, we need to add a premise which says these other scenarios aren't possible. That way, we can know that unemployment will decrease.

Answer (A) does this. It cuts off the possibility that the government will do something with spending that has not been covered by the two premises provided. Once we know that the possible options are only the two provided, the conclusion that unemployment will soon decrease follows logically.

Answer (C) is incorrect because it doesn't address the gap in the argument discussed above. Additionally, it doesn't fit in with the argument as a whole. The argument does not discuss demand for workers, so this is not an assumption the argument is being based on.